Math, asked by rishavshawant3924, 9 months ago

(1+tan theta +sec theta)( 1+cot theta-cosec theta) will be

Answers

Answered by ShuchiRecites
15

Solution:

  • tan∅ = sin∅/cos∅
  • sec∅ = 1/cos∅
  • cot∅ = cos∅/sin∅
  • csc∅ = 1/sin∅

→ 1 + tan∅ + sec∅ = 1 + sin∅/cos∅ + 1/cos∅

→ 1 + tan∅ + sec∅ = (cos∅ + sin∅ + 1)/cos∅

→ 1 + cot∅ + csc∅ = 1 + cos∅/sin∅ - 1/sin∅

→ 1 + cot∅ + csc∅ = (sin∅ + cos∅ - 1)/sin∅

→ (1 + tan∅ + sec∅)/(1 + cot∅ + csc∅)

→ (cos∅ + sin∅ + 1)/cos∅ × sin∅/(sin∅ + cos∅ - 1)

→ tan∅ × (cos∅ + sin∅ + 1)/(sin∅ + cos∅ - 1)

Multiplying by (cos∅ + sin∅ + 1) we get,

→ tan∅ × (cos∅ + sin∅ + 1)²/[(sin∅ + cos∅)² - 1]

→ tan∅ × (cos²∅ + sin²∅ + 1 + 2cos∅ sin∅ + 2 sin∅ + 2 cos∅)/(sin²∅ + cos²∅ + 2sin∅cos∅ - 1)

→ tan∅ × (1 + 1 + 2cos∅ sin∅ + 2sin∅ + 2cos∅)/(1 + 2sin∅cos∅ - 1)

→ tan∅ × 2(1 + sin∅ + cos∅sin∅ + cos∅)/2(sin∅cos∅)

→ tan∅ × [1/sin∅cos∅ + sin∅/sin∅cos∅ + cos∅/sin∅cos∅ + sin∅cos∅/sin∅cos∅]

→ tan∅(sec∅ csc∅ + sec∅ + csc∅ + 1)

Answered by Anonymous
12

\:\:\:\:\:\:\:\:\:\:\:\:\:\:\:\:\:\:\:\huge\boxed{\mathbb{\red{SOLUTION}}}

To Find The Value of

\implies(1 + tan \theta + sec \theta)(1 + cot \theta - cosec \theta)=?

Using ForMuLaS

\bf\Longrightarrow \tan\theta=\frac{\sin\theta}{\cos\theta}\\</p><p></p><p>\bf\Longrightarrow \cot\theta=\frac{\cos\theta}{\sin\theta}\\</p><p></p><p>\bf\Longrightarrow \cosec\theta=\frac{1}{\sin\theta}\\</p><p></p><p>\bf\Longrightarrow \sec\theta=\frac{1}{\cos\theta}\\</p><p>\bf\Longrightarrow \sin{}^{2}\theta+\cos{}^{2}\theta=1\\</p><p></p><p>\bf\Longrightarrow \sec{}^{2}\theta-\tan{}^{2}\theta=1\\</p><p>\bf\Longrightarrow \cosec{}^{2}\theta-\cot{}^{2}\theta=1\\</p><p>

━━━━━━━━━━━━━━━━━━━━━━━━━━━━━

\underline{\red{\mathbb{LET'S \:SOLVE \:THE\: PROBLEM}}}

━━━━━━━━━━━━━━━━━━━━━━━━━━━━━

\implies(1 + tan \theta + sec \theta)(1 + cot \theta - cosec \theta) \\  </p><p></p><p>\implies(1 +  \frac{sin \theta}{cos \theta}  +  \frac{1}{cos \theta} ) (1 +  \frac{cos \theta}{sin \theta}  -  \frac{1}{sin \theta}  \\</p><p></p><p> \implies( \frac{cos \theta + sin \theta + 1}{cos \theta} )( \frac{sin \theta \:  + cos \theta - 1}{sin \theta})  \\</p><p></p><p> \implies\{ \frac{(sin \theta + cos \theta  + 1)(sin \theta + cos \theta - 1)}{sin \theta \: cos \theta} \}\\</p><p></p><p> \implies \frac{ \{(sin \theta +  cos \theta) {}^{2} - 1 {}^{2}   \}}{sin \theta \: cos \theta}  \\ </p><p></p><p> \implies \:  \frac{  \{sin {}^{2} \theta + cos {}^{2}  \theta + 2sin \theta \: cos \theta - 1  \}}{sin \theta \: cos \theta}   \\</p><p></p><p>  \implies \:  \frac{\cancel1 + 2sin  \theta \: cos \theta - \cancel1}{sin \theta \: cos \theta}  \:  \\  </p><p></p><p>\implies \frac{  \: 2 \: \cancel{sin \theta \:  cos  \theta}}{\cancel{sin \theta \: cos  \theta}}  \\ </p><p></p><p> \implies\bf\boxed{\huge{\red{2}}}

━━━━━━━━━━━━━━━━━━━━━━━━━━━━━

\underline{ \huge\mathfrak{hope \: this \: helps \: you}}

\mathcal{ \&amp;#35;\mathcal{answer with quality  }\:  \:  \&amp;#38;  \:  \: \&amp;#35;BAL }

Similar questions